Đến nội dung

Hình ảnh

Số học -Tuyển tập các bài toán sưu tầm từ Mathlinks.ro

- - - - - tuyển tập-sưu tầm.

  • Please log in to reply
Chủ đề này có 44 trả lời

#1
dark templar

dark templar

    Kael-Invoker

  • Hiệp sỹ
  • 3788 Bài viết
Thân chào tất cả các bạn :)

Để thay cho lời mở đầu,mời các bạn đọc qua topic sau.

Nào ,chúng ta cùng bắt tay vào vấn đề chính. :)

Bài toán 1:
Giải hệ phương trình sau :

\[\left\{ \begin{array}{l}
\left\lfloor x \right\rfloor + \left\{ y \right\} = z\\
\left\lfloor y \right\rfloor + \left\{ z \right\} = x\\
\left\lfloor z \right\rfloor + \left\{ x \right\} = y
\end{array} \right.\]

Bài viết đã được chỉnh sửa nội dung bởi dark templar: 11-03-2013 - 17:41

"Do you still... believe in me ?" Sarah Kerrigan asked Jim Raynor - Starcraft II:Heart Of The Swarm.

#2
ducthinh26032011

ducthinh26032011

    Thượng sĩ

  • Thành viên
  • 290 Bài viết

Thân chào tất cả các bạn :)

Để thay cho lời mở đầu,mời các bạn đọc qua topic sau.

Nào ,chúng ta cùng bắt tay vào vấn đề chính. :)

Bài toán 1:
Giải phương trình sau : $\left\lfloor x \right\rfloor + \left\{y \right\}=z$.

Em không hiểu đề toán lắm ạ,nhưng thấy giải ra nó kì kì sao ấy
$[x]+\left\{y \right\}=[z]+\left\{z \right\}$
Giả sử $[x]> [z]$
$\Rightarrow [x]=[z]+a$ với $a\in Z^{+}$
$\Rightarrow [x]+\left\{y \right\}=[z]+\left\{y \right\}+a$
$\Leftrightarrow [z]+\left\{z \right\}=[z]+\left\{y \right\}+a\Leftrightarrow \left\{z \right\}=\left\{y \right\}+a$ (vô lý)
Chứng minh tương tự khi $[z]> [x]$
$\Rightarrow [z]=[x]$ $\Rightarrow \left\{z \right\}=\left\{y \right\}$
Vậy ...

Hình đã gửi


#3
hungchng

hungchng

    Sĩ quan

  • Điều hành viên
  • 337 Bài viết

Thân chào tất cả các bạn :)

Để thay cho lời mở đầu,mời các bạn đọc qua topic sau.

Nào ,chúng ta cùng bắt tay vào vấn đề chính. :)

Bài toán 1:
Giải phương trình sau : $\left\lfloor x \right\rfloor + \left\{y \right\}=z$ với $x,y,z \in \mathbb{Z}$.

$x,y,z \in \mathbb{Z}$ thì phần nguyên $\left\lfloor x \right\rfloor=x$ và phần thập phân $\left\{ y \right\}=0$

Bài viết đã được chỉnh sửa nội dung bởi dark templar: 11-03-2013 - 09:12

Hình đã gửi

#4
hungchng

hungchng

    Sĩ quan

  • Điều hành viên
  • 337 Bài viết

Thân chào tất cả các bạn :)

Để thay cho lời mở đầu,mời các bạn đọc qua topic sau.

Nào ,chúng ta cùng bắt tay vào vấn đề chính. :)

Bài toán 1:
Giải phương trình sau : $\left\lfloor x \right\rfloor + \left\{y \right\}=z$.


Đề như vầy chứ

Giải hệ phương trình $$\begin{cases}[x]+\{y\}=z\\ [y]+\{z\}=x\\ [z]+\{x\}=y\end{cases}$$
Hình đã gửi

#5
ducthinh26032011

ducthinh26032011

    Thượng sĩ

  • Thành viên
  • 290 Bài viết

Trong file mà Nam (zipienie) gửi em thì đề chỉ cho là phương trình thôi thầy.Để em gửi tin nhắn bài giải cho thầy xem đúng hay sai.

Sau khi bàn luận với thầy Hùng,mình xin sửa đề thành hệ phương trình như thầy Hùng đã đề nghị. Thành thật xin lỗi các bạn vì không kiểm tra kỹ đề. :(

Nếu thế thì giải như em làm bên trên rồi làm tương tự với các phương trình còn lại,suy ra $x=y=z$
@@ Như thế có đúng k ạ?

Hình đã gửi


#6
dark templar

dark templar

    Kael-Invoker

  • Hiệp sỹ
  • 3788 Bài viết

Nếu thế thì giải như em làm bên trên rồi làm tương tự với các phương trình còn lại,suy ra $x=y=z$
@@ Như thế có đúng k ạ?

Đúng rồi đấy em :)

1 câu hỏi mở là liệu có thể giải được hệ tổng quát sau hay không ?

Tổng quát bài toán 1: Giải hệ phương trình sau :
\[\left\{ \begin{array}{l}
\left\lfloor {{x_1}} \right\rfloor + \left\{ {{x_2}} \right\} = {x_3} + {x_4} + ... + {x_n}\\
\left\lfloor {{x_2}} \right\rfloor + \left\{ {{x_3}} \right\} = {x_4} + {x_5} + ... + {x_n} + {x_1}\\
...\\
\left\lfloor {{x_n}} \right\rfloor + \left\{ {{x_1}} \right\} = {x_2} + {x_3} + ... + {x_{n - 1}}
\end{array} \right.\]
"Do you still... believe in me ?" Sarah Kerrigan asked Jim Raynor - Starcraft II:Heart Of The Swarm.

#7
dark templar

dark templar

    Kael-Invoker

  • Hiệp sỹ
  • 3788 Bài viết

Đúng rồi đấy em :)

1 câu hỏi mở là liệu có thể giải được hệ tổng quát sau hay không ?

Tổng quát bài toán 1: Giải hệ phương trình sau :
\[\left\{ \begin{array}{l}
\left\lfloor {{x_1}} \right\rfloor + \left\{ {{x_2}} \right\} = {x_3} + {x_4} + ... + {x_n}\\
\left\lfloor {{x_2}} \right\rfloor + \left\{ {{x_3}} \right\} = {x_4} + {x_5} + ... + {x_n} + {x_1}\\
...\\
\left\lfloor {{x_n}} \right\rfloor + \left\{ {{x_1}} \right\} = {x_2} + {x_3} + ... + {x_{n - 1}}
\end{array} \right.\]

Chúng ta còn 1 câu hỏi mở ở đây.Mong mọi người có ý kiến. Đáp án của ducthinh hoàn toàn giống với đáp án mình có.

Và... bắt đầu qua đề mới :

Bài toán 2: Chứng minh rằng $\sum\limits_{k = 0}^n {\frac{{n!}}{{(n - k)!}}} = \left\lfloor {n!e} \right\rfloor \quad \forall n>0$

Bài toán 3: Tìm tất cả các số nguyên tố thỏa mãn $p + {q^2} + {r^3} = 200$.

Nào,mời các bạn chém và thảo luận cho ý kiến. :)

Bài viết đã được chỉnh sửa nội dung bởi dark templar: 14-03-2013 - 17:44

"Do you still... believe in me ?" Sarah Kerrigan asked Jim Raynor - Starcraft II:Heart Of The Swarm.

#8
WhjteShadow

WhjteShadow

    Thượng úy

  • Phó Quản lý Toán Ứng dụ
  • 1323 Bài viết

Chúng ta còn 1 câu hỏi mở ở đây.Mong mọi người có ý kiến. Đáp án của ducthinh hoàn toàn giống với đáp án mình có.

Và... bắt đầu qua đề mới :

Bài toán 2: Chứng minh rằng $\sum\limits_{k = 0}^n {\frac{{n!}}{{(n - k)!}}} = \left\lfloor {n!e} \right\rfloor \quad \forall n>0$

Bài toán 3: Tìm tất cả các số nguyên tố thỏa mãn $p + {q^2} + {r^3} = 200$.

Nào,mời các bạn chém và thảo luận cho ý kiến. :)

Bài 2.
Để ý đẳng thức quen thuộc $e=1+\frac{1}{1!}+\frac{1}{2!}+...+\frac{1}{a!}$ với $a\to \infty$ (Chứng minh bằng cách xét khai triển MacLaurin của hàm $e^{x}$). Ta có :
$$n!e=n!+n!+\frac{n!}{2!}+\frac{n!}{3!}+...+\frac{n!}{n!}+\frac{n!}{(n+1)!}+\frac{n!}{(n+2)!}+.....$$
$$=\sum\limits_{k = 0}^n {\frac{{n!}}{{(n - k)!}}} +\frac{n!}{(n+1)!}+\frac{n!}{(n+2)!}+.....$$
Vậy nên bài toán sẽ hoàn tất nếu ta chỉ ra được :")
$$\frac{n!}{(n+1)!}+\frac{n!}{(n+2)!}+.....<1$$
Hay là:
$$\frac{1}{n+1}+\frac{1}{(n+1)(n+2)}+\frac{1}{(n+1)(n+2)(n+3)}+....<1$$
Để ý $\frac{1}{(n+1)(n+2)}=\frac{1}{n+1}-\frac{1}{n+2}\\ \frac{1}{(n+1)(n+2)(n+3)}<\frac{1}{(n+2)(n+3)}=\frac{1}{n+2}-\frac{1}{n+3}\\\frac{1}{(n+1)(n+2)(n+3)(n+4)}<\frac{1}{(n+3)(n+4)}=\frac{1}{n+3}-\frac{1}{n+4}....$
Vậy nên
$$\frac{n!}{(n+1)!}+\frac{n!}{(n+2)!}+.....<\frac{2}{n+1}<1$$
Hoàn tất chứng minh !
Đề xuất bài toán mở rộng :
Mở rộng bài toán 2:
Ch0 dãy số $\{x_n\}$, $x_1=1,x_{n+1}=\left\lfloor {(x_n)!e} \right\rfloor \quad$
Chứng minh rằng $(x_m;x_n)=x_{(m;n)}$
Bài toán 3.
Bài này xét trâu là ra :))
Do $r^3< 200$ nên $r\in \{2;3;5\}$

$\bullet$ nếu $r=5$ thì $p+q^2=75$ suy ra tr0ng $p,q$ phải có 1 số chẵn và bằng 2, thử vào ta có đáp số $q=2,p=71$

$\bullet$ nếu $r=3$ thì $p+q^2=173$ suy ra tr0ng $p,q$ phải có 1 số chẵn và bằng 2, thử vào ta k có kết quả snt, loại !
$\bullet$ nếu $r=2$ thì $p+q^2=192$, $p,q$ cùng tính chẵn lẻ nhưng 2 số không thể cùng =2 nên cả 2 số đều lẻ. $p^2\leq 192$ nên $q\in \{3;5;7;11;13\}$, thử vào ta có $(p;q)=(167;5),(71;11),(23,13)$
Vậy $(p;q;r)=(2;3;5),(167;5;2),(71;11;2),(23,13;2)$
----------
Dạ e sửa rùi :))

Bài viết đã được chỉnh sửa nội dung bởi WhjteShadow: 16-03-2013 - 21:36

“There is no way home, home is the way.” - Thich Nhat Hanh

#9
dark templar

dark templar

    Kael-Invoker

  • Hiệp sỹ
  • 3788 Bài viết

Vậy $(p;q;r)=(2;3;5),(167;5;2),(143;7;2),(71;11;2),(23,13;2)$

$143$ chia hết cho 13 mà em,sao là số nguyên tố được em,nhưng nhìn chung làm đúng. :D
"Do you still... believe in me ?" Sarah Kerrigan asked Jim Raynor - Starcraft II:Heart Of The Swarm.

#10
dark templar

dark templar

    Kael-Invoker

  • Hiệp sỹ
  • 3788 Bài viết
Tiếp tục nào :)

Bài toán 4: Tìm tất cả các số nguyên $m$ sao cho $m^4+m^3+1$ là 1 số chính phương.

Bài toán 5: Cho $x = - 2272,y = {10^3} + {10^2}c + 10b + a,z = 1$ thỏa mãn phương trình $ax + by + cz = 1$,trong đó $a,b,c$ là các số nguyên và $a<b<c$.Tìm $y$.
"Do you still... believe in me ?" Sarah Kerrigan asked Jim Raynor - Starcraft II:Heart Of The Swarm.

#11
Idie9xx

Idie9xx

    Sĩ quan

  • Thành viên
  • 319 Bài viết

Tiếp tục nào :)

Bài toán 4: Tìm tất cả các số nguyên $m$ sao cho $m^4+m^3+1$ là 1 số chính phương.
 

Ta có $m^4+m^3+1=m^3(m+1)+1=k^2$ dễ thấy $k$ lẻ.

Biển đổi thành $m^3(m+1)=(k-1)(k+1)$ mà $(k-1;k+1)=2$ và $(m;m+1)=1$ nên $m=m(m+1)-m^2=(k+1)-(k-1)=2$

Vậy số nguyên thỏa mãn là $m=2$ :D


Bài viết đã được chỉnh sửa nội dung bởi Idie9xx: 19-03-2013 - 13:12

$\large \circ \ast R_f\cdot Q_r\cdot 1080\ast \circ$

#12
dark templar

dark templar

    Kael-Invoker

  • Hiệp sỹ
  • 3788 Bài viết

@Idie9xx: Làm thiếu nghiệm rồi bạn,bài này không phải tìm $m$ nguyên dương không thôi đâu.

Tiếp tục nào :)

Bài toán 4: Tìm tất cả các số nguyên $m$ sao cho $m^4+m^3+1$ là 1 số chính phương.

Bài toán 5: Cho $x = - 2272,y = {10^3} + {10^2}c + 10b + a,z = 1$ thỏa mãn phương trình $ax + by + cz = 1$,trong đó $a,b,c$ là các số nguyên và $a<b<c$.Tìm $y$.

 

Bài toán 4 là 1 bài toán quen thuộc,có thể sử dụng chặn bình phương:)

 

Lời giải bài toán 4: 

Đặt ${m^4} + {m^3} + 1 = {n^2} \quad (n \in \mathbb{Z}) \Leftrightarrow 64{m^4} + 64{m^3} + 64 = {(8n)^2}$

$ \Leftrightarrow {(8{m^2} + 4m - 1)^2} + 8m + 63 = {(8n)^2}$.

 

Nếu $m=0$ thì rõ ràng thỏa mãn.

 

Nếu $m \neq 0$ và $m>-8$ thì ${(8{m^2} + 4m - 1)^2} + 8m + 63 = {(8n)^2} \Rightarrow {(8n)^2} > {(8{m^2} + 4m - 1)^2}$.

Do $8m^2-4m+1 \ge 0$ nên ${(8n)^2} \ge {(8{m^2} + 4m)^2}$

Vậy ${(8{m^2} + 4m - 1)^2} + 8m + 63 \ge {(8{m^2} + 4m)^2}$

Vì thế ${m^2} \le 4 \Rightarrow m \in \{  - 2, - 1,1,2\} $

 

Nếu $m \le -8$ thì ${(8{m^2} + 4m - 1)^2} + 8m + 63 = {(8n)^2} \Rightarrow {(8n)^2} < {(8{m^2} + 4m - 1)^2}$

Do $8m^2-4m+1\ge 0$ nên ${(8n)^2} \le {(8{m^2} + 4m - 2)^2}$

Vậy ${(8{m^2} + 4m - 1)^2} + 8m + 63 \le {(8{m^2} + 4m - 2)^2}$ và $4{m^2} + 4m + 15 \le 0$ (vô lý).

 

Từ đó ta tìm được các nghiệm là $\boxed{m \in \{  - 2, - 1,0,2\} }$.

 

Lời giải bài toán 5:

Từ gỉa thuyết,ta có $b(1000 + 100c + 10b + a) + c = 2272a + 1 \quad (*)$

 

Vì $b + 1 \le c;1 \le a$ và $a \le b-1$ nên suy ra :

 

$$\begin{array}{l}b(1000 + 100(b + 1) + 10b + 1) + b + 1 \le 2272(b - 1) + 1\\\Leftrightarrow 110{b^2} - 1170b + 2272 \le 0\\\Leftrightarrow 3 \le b \le 8\end{array}$$
 
Nếu $b=8$ thì PT $(*)$ trở thành $8639 + 801c = 2264a$ chỉ có nghiệm với $1 \le a \le 7$.Thử trực tiếp thấy $a=7;c=9$ thỏa mãn.
 
Nếu $b=7$ thì PT $(*)$ trở thành $7489 + 701c = 2265a$ không có nghiệm với $1 \le a \le 6$.
 
Nếu $b=6$ thì PT $(*)$ trở thành $6359 + 601c = 2266a$ không có nghiệm với $1 \le a \le 5$.
 
Nếu $b=5$ thì PT $(*)$ trở thành $5249 + 501c = 2267a$ không có nghiệm với $1 \le a \le 4$.
 
Nếu $b=4$ thì PT $(*)$ trở thành $4159 + 401c = 2268a$ không có nghiệm với $1 \le a \le 3$.
 
Nếu $b=3$  thì PT $(*)$ trở thành $3089 + 301c = 2269a$ cũng không có nghiệm với $1 \le a \le 2$.
 
Vậy chỉ có duy nhất nghiệm là $(a;b;c)=(7;8;9)$ và $y=1987$.
 
**********

Và chúng ta có 2 câu hỏi mở sau ,mong các bạn cho ý kiến. :)

Tổng quát bài toán 1: Giải hệ phương trình sau :
\[\left\{ \begin{array}{l}
\left\lfloor {{x_1}} \right\rfloor + \left\{ {{x_2}} \right\} = {x_3} + {x_4} + ... + {x_n}\\
\left\lfloor {{x_2}} \right\rfloor + \left\{ {{x_3}} \right\} = {x_4} + {x_5} + ... + {x_n} + {x_1}\\
...\\
\left\lfloor {{x_n}} \right\rfloor + \left\{ {{x_1}} \right\} = {x_2} + {x_3} + ... + {x_{n - 1}}
\end{array} \right.\]

 

 

Mở rộng bài toán 2:
Ch0 dãy số $\{x_n\}$, $x_1=1,x_{n+1}=\left\lfloor {(x_n)!e} \right\rfloor \quad$
Chứng minh rằng $(x_m;x_n)=x_{(m;n)}$


Bài viết đã được chỉnh sửa nội dung bởi dark templar: 24-03-2013 - 09:30

"Do you still... believe in me ?" Sarah Kerrigan asked Jim Raynor - Starcraft II:Heart Of The Swarm.

#13
dark templar

dark templar

    Kael-Invoker

  • Hiệp sỹ
  • 3788 Bài viết

Đề mới :

 

Bài toán 6: Tìm nghiệm tự nhiên thỏa $\frac{1}{x} + \frac{2}{y} - \frac{3}{z} = 1$.

 

Bài toán 7: Chia tập số nguyên dương thành các nhóm như sau :

 

Nhóm thứ nhất có một số: 1

Nhóm thứ hai có hai số:  2;3

Nhóm thứ ba có ba số:  4;5;6

Nhóm thứ tư có bốn số: 7;8;9;10

...

Cứ tiếp tục như vậy.Gọi $S(k)$ là tổng các số trong nhóm thứ $k$.Tính tổng $\sum_{i=1}^{n}S(2i-1)$.


Bài viết đã được chỉnh sửa nội dung bởi dark templar: 01-06-2013 - 21:00
Chèn link !

"Do you still... believe in me ?" Sarah Kerrigan asked Jim Raynor - Starcraft II:Heart Of The Swarm.

#14
nguyenthehoan

nguyenthehoan

    Sĩ quan

  • Thành viên
  • 392 Bài viết

Đề mới :

 

Bài toán 6: Tìm nghiệm tự nhiên thỏa $\frac{1}{x} + \frac{2}{y} - \frac{3}{z} = 1$.

 

Bài toán 7: Chia tập số nguyên dương thành các nhóm như sau :

 

Nhóm thứ nhất có một số: 1

Nhóm thứ hai có hai số:  2;3

Nhóm thứ ba có ba số:  4;5;6

Nhóm thứ tư có bốn số: 7;8;9;10

...

Cứ tiếp tục như vậy.Gọi $S(k)$ là các số trong nhóm thứ $k$.Tính tổng $\sum_{i=1}^{n}S(2i-1)$.

Ta xét các số hạng đầu mỗi dòng.Gọi $a_{i}$ là số hạng đầu tiên nhóm thứ i.

 

Dễ dàng chứng minh băng quy nạp 

 

$a_{i}=\frac{n(n-1)}{2}+1$

 

Tại dòng i có i số hạng nên

 

$S(i)=[\frac{n(n-1)}{2}+1]+[\frac{n(n-1)}{2}+2]+...+[\frac{n(n-1)}{2}+n]$

 

$=\frac{n^{2}(n-1)}{2}+\frac{n(n+1)}{2}=\frac{n(n^{2}+1)}{2}$

 

Tiếp theo ta sẽ cm bằng quy nạp

 

$S_{1}+S_{3}+...+S_{2n-1}=n^{4}$

 

Thật vậy

 

Với n=1,hiển nhiên đúng.

 

Giả sử $S_{1}+S_{3}+...+S_{2k-1}=k^{4}$

 

Ta có $S_{1}+S_{3}+...+S_{2k-1}+S_{2(k+1)-1}=n^{4}+\frac{(2n+1)((2n+1)^{2}+1)}{2}=(n+1)^{4}$

 

Vậy giả thiết đúng tới k+1.

 

Ta có dpcm.

 


Bài viết đã được chỉnh sửa nội dung bởi nguyenthehoan: 24-03-2013 - 12:01


#15
dark templar

dark templar

    Kael-Invoker

  • Hiệp sỹ
  • 3788 Bài viết

Bài 7 anh định nghĩa lại $S(k)$ đi ạ???

Àh,một chút lầm lẫn :P $S(k)$ là tổng các số trong nhóm $k$.


"Do you still... believe in me ?" Sarah Kerrigan asked Jim Raynor - Starcraft II:Heart Of The Swarm.

#16
dark templar

dark templar

    Kael-Invoker

  • Hiệp sỹ
  • 3788 Bài viết

Bài toán 6: Tìm nghiệm tự nhiên thỏa $\frac{1}{x} + \frac{2}{y} - \frac{3}{z} = 1$.

Lời giải bài toán 6:

PT đã cho trở thành viết lại thành $\frac{1}{x} + \frac{2}{y} = 1 + \frac{3}{z}$.

 

Nếu $x = 1 \Rightarrow 3y = 2z$ và các nghiệm là $(x,y,z) = (1,2t,3t) \quad t \in \mathbb{N}$.

 

Xét $x>1$ thì từ PT ta suy ra $\frac{1}{x} + \frac{2}{y} > 1 \Rightarrow y < 2\left( {1 + \frac{1}{{x - 1}}} \right)$.

 

Nếu $x=2$ thì $y<4$ khi đó thì $y \in \{ 1,2,3\} $ và nghiệm của PT là $(x,y,z) \in \{ (2,1,2);(2,2,6);(2,3,18)\} $.

 

Nếu $x \ge 3$ thì $y<3$ ,khi đó $y \in \{ 1,2\} $.

 

Với $y = 1 \Rightarrow z = \frac{{3x}}{{x + 1}}$ ,vậy thì $x + 1|3$ ,vô lý vì $x \ge 3$.

 

Với $y = 2 \Rightarrow z = 3x$ và nghiệm của phương trình là $(x,y,z) = (t,2,3t) \quad t \in \mathbb{N}$.

 

Vậy các nghiệm của phương trình là $\boxed{(x,y,z) \in \{ (2,1,2);(2,3,18);(1,2t,3t);(t,2,3t)\} }$,trong đó $t \in \mathbb{N}$.

 

**********

Đề mới:

 

Bài toán 8: Cho số nguyên $m>3$.Chứng minh rằng $\left\lfloor {\frac{{m(m + 1)}}{{2(2m - 1)}}} \right\rfloor  = \left\lfloor {\frac{{m + 1}}{4}} \right\rfloor $.

 

Bài toán 9: Giải hệ PT $\left\{ {\begin{array}{*{20}{c}}{\left\lfloor x \right\rfloor  + \{ y\}  = \left\lfloor y \right\rfloor \{ x\} }\\{x + y = n}\end{array}} \right.$


"Do you still... believe in me ?" Sarah Kerrigan asked Jim Raynor - Starcraft II:Heart Of The Swarm.

#17
hxthanh

hxthanh

    Tín đồ $\sum$

  • Hiệp sỹ
  • 3915 Bài viết

**********
Đề mới:
 
Bài toán 8: Cho số nguyên $m>3$.Chứng minh rằng $\left\lfloor {\frac{{m(m + 1)}}{{2(2m - 1)}}} \right\rfloor  = \left\lfloor {\frac{{m + 1}}{4}} \right\rfloor $.
...

 

Vì $m\ge 4$ là số nguyên nên cách đơn giản nhất là đưa về tập số dư

 

Đặt $m=4k+\{0,1,2,3\}\quad(k\ge 1)$, tương ứng với 4 số dư, ta có:

 

$VP=\left\lfloor\dfrac{m+1}{4}\right\rfloor=\left\lfloor\dfrac{4k+\{1,2,3,4\}}{4}\right\rfloor=k+\{0,0,0,1\}$

 

$VT=\left\lfloor\dfrac{m(m+1)}{4m-2}\right\rfloor=\left\lfloor\dfrac{(4k+\{0,1,2,3\})(4k+\{1,2,3,4\})}{4(4k+\{0,1,2,3\})-\{2,2,2,2\}}\right\rfloor$

$=\left\lfloor\dfrac{16k^2+k\{4,8,12,16\}+k\{0,4,8,12\}+\{0,2,6,12\}}{16k+\{-2,2,6,10\}}\right\rfloor$

$=\left\lfloor k+\{0,0,0,1\}+\dfrac{k\{6,10,14,2\}+\{0,2,6,2\}}{16k+\{-2,2,6,10\}}\right\rfloor$

$=k+\{0,0,0,1\}+\left\lfloor\dfrac{\{6k,10k+2,14k+6,2k+2\}}{\{16k-2,16k+2,16k+6,16k+10\}}\right\rfloor$

 

Vì $k\ge 1$ nên $\begin{cases}6k&<&16k-2\\10k+2&<&16k+2\\14k+6&<&16k+6\\2k+2&<&16k+10\end{cases}$

 

Do đó $\left\lfloor\dfrac{\{6k,10k+2,14k+6,2k+2\}}{\{16k-2,16k+2,16k+6,16k+10\}}\right\rfloor=0$

 

Từ đó suy ra điều phải chứng minh :)



#18
dark templar

dark templar

    Kael-Invoker

  • Hiệp sỹ
  • 3788 Bài viết

Bài toán 8: Cho số nguyên $m>3$.Chứng minh rằng $\left\lfloor {\frac{{m(m + 1)}}{{2(2m - 1)}}} \right\rfloor  = \left\lfloor {\frac{{m + 1}}{4}} \right\rfloor $.

 

Bài toán 9: Giải hệ PT $\left\{ {\begin{array}{*{20}{c}}{\left\lfloor x \right\rfloor  + \{ y\}  = \left\lfloor y \right\rfloor \{ x\} }\\{x + y = n}\end{array}} \right.$

Lời giải bài toán 8:

$\frac{{m(m + 1)}}{{2(2m - 1)}} = \frac{{m + 1}}{4} + \frac{{m + 1}}{{4(2m - 1)}}$ với $m \ge 3$ thì $\frac{{m + 1}}{{4(2m - 1)}} < \frac{1}{4}$.

 

 

Vậy $\frac{{m + 1}}{4} < \frac{{m(m + 1)}}{{2(2m - 1)}} = \frac{{m + 1}}{4} + a$ với $0 < a < \frac{1}{4}$.

 

Vậy rõ ràng là $\left\lfloor {\frac{{m + 1}}{4} + a} \right\rfloor  = \left\lfloor {\frac{{m + 1}}{4}} \right\rfloor $ khi $0<a<\frac{1}{4}$.

 

Lời giải bài toán 9:

Đặt $x = p + u$ với $p \in \mathbb{Z}$ và $u \in [0;1)$,khi đó từ PT thứ 2 của hệ suy ra $y = n - p - u$.

 

Nếu $u=0$ thì ta có $\left\{ \begin{array}{l}\left\lfloor x \right\rfloor  = p\\\{ x\}  = 0\end{array} \right.$ và $\left\{\begin{array}{l}\left\lfloor y \right\rfloor  = n - p\\\{ y\}  = 0\end{array} \right.$

 

Khi đó PT đầu tiên của hệ trở thành $p=0$.

 

Nếu $u \neq 0$ thì ta có $\left\{ \begin{array}{l}\left\lfloor x \right\rfloor  = p\\\{ x\}  = u\end{array} \right.$ và $\left\{ \begin{array}{l}\left\lfloor y \right\rfloor  = n - p - 1\\\{ y\}  = 1 - u\end{array} \right.$.

 

Khi đó PT đầu tiên của hệ trở thành $p + 1 = (n - p)u \quad (*)$.

 

  • Nếu $p=n$ thì PT $(*)$ trở thành $n = p =  - 1 \implies n \in \mathbb{N}$.
  • Nếu $p>n$ thì từ PT $(*)$ suy ra $p + 1 < 0$ và $n < p <  - 1$,vô lý vì $n \in \mathbb{N}$.

 

 

Vậy $p<n$,$p + 1 > 0$ và $p + 1 < n - p$ và suy ra $0 \le p < \frac{{n - 1}}{2}$.

 

Từ đó ta tìm được các nghiệm của hệ là $(x,y) = (0,n)$.

 

Nếu $n>1$ thì $(x,y) = \left( {p + \frac{{p + 1}}{{n - p}},n - p - \frac{{p + 1}}{{n - p}}} \right)$ với mọi số nguyên $p \in \left[ {0,\frac{{n - 1}}{2}} \right)$.

 

**********

Đề mới:

 

Bài toán 10: Giải PT $x = 8\left\lfloor {\sqrt[4]{x}} \right\rfloor  + 3$ trên tập số tự nhiên.

 

Bài toán 11: Giải PT $\left\lfloor {{x^3}} \right\rfloor  + \left\lfloor {{x^2}} \right\rfloor  + \left\lfloor x \right\rfloor  = \{ \;{x^{1994}}\}  - 1$.


"Do you still... believe in me ?" Sarah Kerrigan asked Jim Raynor - Starcraft II:Heart Of The Swarm.

#19
Mai Xuan Son

Mai Xuan Son

    Vagrant

  • Thành viên
  • 274 Bài viết


 

 

Bài toán 10: Giải PT $x = 8\left\lfloor {\sqrt[4]{x}} \right\rfloor  + 3$ trên tập số tự nhiên.

 

Bài 10 giới hạn miền là ok

Đặt $\sqrt[4]{x}=[\sqrt[4]{x}]+\left \{ \sqrt[4]{x} \right \}=a+b$

Với $b=\left \{ \sqrt[4]{x} \right \}\in [0;1)$

$a=[\sqrt[4]{x}]$

Phương trình tương đương với $1\geq b=\sqrt[4]{8a+3}-a> 0$

Từ đó ta được $a\in \left \{ 0;1;2 \right \}$

Thử từng trường hợp, thấy thoả, kết luận

$a\in \left \{ 0;1;2 \right \}$

Do đó $x\in \left \{ 3;11;19 \right \}$


Bài viết đã được chỉnh sửa nội dung bởi Mai Xuan Son: 14-04-2013 - 21:44

~~~like phát~~~

#20
dark templar

dark templar

    Kael-Invoker

  • Hiệp sỹ
  • 3788 Bài viết

Bài toán 11: Giải PT $\left\lfloor {{x^3}} \right\rfloor  + \left\lfloor {{x^2}} \right\rfloor  + \left\lfloor x \right\rfloor  = \{ \;{x^{1994}}\}  - 1$.

Bài 10 Sơn đã giải đúng rồi đó :)

**********

Lời giải bài toán 11:

Vì $u \ge \left\lfloor u \right\rfloor  > u - 1$ nên ta có ${x^3} + {x^2} + x \ge VT > {x^3} + {x^2} + x - 3$.

 

Vì $VP \in [ - 1,0)$ nên ta có ${x^3} + {x^2} + x \ge  - 1$ và ${x^3} + {x^2} + x - 3 < 0$,do đó suy ra $3 > {x^3} + {x^2} + x \ge  - 1$.

 

Vì ${x^3} + {x^2} + x$ là hàm số đồng biến trên $\mathbb{R}$ nên suy ra $1 > x \ge  - 1$.

 

Nếu $x \in [0,1)$ thì $VT = 0$ và $VP = {x^{1994}} - 1$, nên ta có PT ${x^{1994}} = 1$.Rõ ràng là PT này vô nghiệm trong $[0,1)$.

 

Nếu $x \in ( - 1,0)$ thì $VT =  - 1 + 0 - 1 =  - 2$ và $VP = {x^{1994}} - 1$,khi đó ta có PT ${x^{1994}} =  - 1$.Rõ ràng là PT này cũng vô nghiệm trong $( - 1,0)$.

 

Nếu $x =  - 1$ thì $VT = VP =  - 1$.

 

Vậy $\boxed{\displaystyle x=-1}$ là nghiệm duy nhất.

 

**********

Đề mới:

 

Bài toán 12: Giải PT $\left\lfloor {{x^2} + 1} \right\rfloor  = \left\lfloor {2x} \right\rfloor $.

 

Bài toán 13: Tìm tất cả các nghiệm thực của PT $\left( {1 - \left\lfloor a \right\rfloor } \right)\left\lfloor {\frac{a}{{1 - a}}} \right\rfloor  = \left\lfloor a \right\rfloor $.


"Do you still... believe in me ?" Sarah Kerrigan asked Jim Raynor - Starcraft II:Heart Of The Swarm.





Được gắn nhãn với một hoặc nhiều trong số những từ khóa sau: tuyển tập-sưu tầm.

1 người đang xem chủ đề

0 thành viên, 1 khách, 0 thành viên ẩn danh